Das Wasserstoffatom (relativistsich): Difference between revisions

From testwiki
Jump to navigation Jump to search
*>SchuBot
m Interpunktion, replaced: ! → ! (5)
 
(2 intermediate revisions by 2 users not shown)
Line 1: Line 1:
{{Scripthinweis|Quantenmechanik|7|5}}
<noinclude>{{Scripthinweis|Quantenmechanik|7|5}}</noinclude>


In einem rotationssymmetrischen Potenzial haben wir als Dirac- Hamiltonian:
In einem rotationssymmetrischen Potenzial haben wir als Dirac- Hamiltonian:


<math>\begin{align}
:<math>\begin{align}


& H=\left( c\bar{\alpha }\bar{p}+{{m}_{0}}{{c}^{2}}\beta +V(r) \right) \\
& H=\left( c\bar{\alpha }\bar{p}+{{m}_{0}}{{c}^{2}}\beta +V(r) \right) \\
Line 21: Line 21:
Man kann den Hamilton- Operator schreiben als:
Man kann den Hamilton- Operator schreiben als:


<math>H=\left( c{{\alpha }_{r}}{{p}_{r}}+\frac{ic}{r}{{\alpha }_{r}}\beta \hbar Q+{{m}_{0}}{{c}^{2}}\beta +V(r) \right)</math>
:<math>H=\left( c{{\alpha }_{r}}{{p}_{r}}+\frac{ic}{r}{{\alpha }_{r}}\beta \hbar Q+{{m}_{0}}{{c}^{2}}\beta +V(r) \right)</math>


Beweis:
Beweis:


<math>\begin{align}
:<math>\begin{align}


& {{\alpha }_{r}}{{p}_{r}}+\frac{i}{r}{{\alpha }_{r}}\beta \hbar Q={{\alpha }_{r}}\left[ \frac{1}{r}\left( \bar{r}\bar{p}-i\hbar  \right)+\frac{i}{r}{{\beta }^{2}}\left( \tilde{\bar{\sigma }}\bar{L}+\hbar  \right) \right] \\
& {{\alpha }_{r}}{{p}_{r}}+\frac{i}{r}{{\alpha }_{r}}\beta \hbar Q={{\alpha }_{r}}\left[ \frac{1}{r}\left( \bar{r}\bar{p}-i\hbar  \right)+\frac{i}{r}{{\beta }^{2}}\left( \tilde{\bar{\sigma }}\bar{L}+\hbar  \right) \right] \\
Line 41: Line 41:
Es gilt weiter:
Es gilt weiter:


<math>\left[ \hbar Q,H \right]=0</math>
:<math>\left[ \hbar Q,H \right]=0</math>
 
.
. Somit existieren gemeinsame Eigenzustände zu <math>H</math>
Somit existieren gemeinsame Eigenzustände zu <math>H</math>


und <math>\hbar Q</math>
und <math>\hbar Q</math>
Line 51: Line 51:
:
:


<math>\begin{align}
:<math>\begin{align}


& {{\left( \hbar Q \right)}^{2}}=\beta \left( \tilde{\bar{\sigma }}\bar{L}+\hbar  \right)\beta \left( \tilde{\bar{\sigma }}\bar{L}+\hbar  \right)={{\beta }^{2}}{{\left( \tilde{\bar{\sigma }}\bar{L}+\hbar  \right)}^{2}} \\
& {{\left( \hbar Q \right)}^{2}}=\beta \left( \tilde{\bar{\sigma }}\bar{L}+\hbar  \right)\beta \left( \tilde{\bar{\sigma }}\bar{L}+\hbar  \right)={{\beta }^{2}}{{\left( \tilde{\bar{\sigma }}\bar{L}+\hbar  \right)}^{2}} \\
Line 77: Line 77:
Somit:
Somit:


<math>\begin{align}
:<math>\begin{align}


& {{\left( \hbar Q \right)}^{2}}={{L}^{2}}+\hbar \tilde{\bar{\sigma }}\bar{L}+{{\hbar }^{2}}={{\left( \bar{L}+\frac{\hbar }{2}\tilde{\bar{\sigma }} \right)}^{2}}+\frac{{{\hbar }^{2}}}{4} \\
& {{\left( \hbar Q \right)}^{2}}={{L}^{2}}+\hbar \tilde{\bar{\sigma }}\bar{L}+{{\hbar }^{2}}={{\left( \bar{L}+\frac{\hbar }{2}\tilde{\bar{\sigma }} \right)}^{2}}+\frac{{{\hbar }^{2}}}{4} \\
Line 91: Line 91:
Schließlich also
Schließlich also


<math>{{\left( \hbar Q \right)}^{2}}={{\bar{J}}^{2}}+\frac{{{\hbar }^{2}}}{4}</math>
:<math>{{\left( \hbar Q \right)}^{2}}={{\bar{J}}^{2}}+\frac{{{\hbar }^{2}}}{4}</math>


Die Eigenwerte von <math>{{\bar{J}}^{2}}</math>
Die Eigenwerte von <math>{{\bar{J}}^{2}}</math>
Line 99: Line 99:
mit <math>j=l\pm s=\frac{1}{2},\frac{3}{2},...</math>
mit <math>j=l\pm s=\frac{1}{2},\frac{3}{2},...</math>


<math>\begin{align}
:<math>\begin{align}


& {{\left( \hbar Q \right)}^{2}}\left| j \right\rangle =\left( {{\hbar }^{2}}j(j+1)+\frac{{{\hbar }^{2}}}{4} \right)\left| j \right\rangle ={{\hbar }^{2}}{{(j+\frac{1}{2})}^{2}}\left| j \right\rangle  \\
& {{\left( \hbar Q \right)}^{2}}\left| j \right\rangle =\left( {{\hbar }^{2}}j(j+1)+\frac{{{\hbar }^{2}}}{4} \right)\left| j \right\rangle ={{\hbar }^{2}}{{(j+\frac{1}{2})}^{2}}\left| j \right\rangle  \\
Line 109: Line 109:
Somit:
Somit:


<math>\begin{align}
:<math>\begin{align}


& \left( \hbar Q \right)\left| j \right\rangle =\left( \hbar q \right)\left| j \right\rangle  \\
& \left( \hbar Q \right)\left| j \right\rangle =\left( \hbar q \right)\left| j \right\rangle  \\
Line 119: Line 119:
Es bleibt das radiale Eigenwertproblem für
Es bleibt das radiale Eigenwertproblem für


<math>H=\left( c{{\alpha }_{r}}{{p}_{r}}+\frac{ic}{r}{{\alpha }_{r}}\beta \hbar Q+{{m}_{0}}{{c}^{2}}\beta +V(r) \right)</math>
:<math>H=\left( c{{\alpha }_{r}}{{p}_{r}}+\frac{ic}{r}{{\alpha }_{r}}\beta \hbar Q+{{m}_{0}}{{c}^{2}}\beta +V(r) \right)</math>


'''Geeignete Darstellung für '''<math>{{\alpha }_{r}}</math>
'''Geeignete Darstellung für '''<math>{{\alpha }_{r}}</math>
Line 125: Line 125:
:
:


<math>\begin{align}
:<math>\begin{align}


& {{\left( {{\alpha }_{r}} \right)}^{2}}=\frac{1}{{{r}^{2}}}\left( \bar{\alpha }\bar{r} \right)\left( \bar{\alpha }\bar{r} \right)=\frac{1}{{{r}^{2}}}{{\alpha }^{\mu }}{{\alpha }^{\nu }}{{x}^{\mu }}{{x}^{\nu }}=\frac{1}{2{{r}^{2}}}\left( {{\alpha }^{\mu }}{{\alpha }^{\nu }}+{{\alpha }^{\nu }}{{\alpha }^{\mu }} \right){{x}^{\mu }}{{x}^{\nu }} \\
& {{\left( {{\alpha }_{r}} \right)}^{2}}=\frac{1}{{{r}^{2}}}\left( \bar{\alpha }\bar{r} \right)\left( \bar{\alpha }\bar{r} \right)=\frac{1}{{{r}^{2}}}{{\alpha }^{\mu }}{{\alpha }^{\nu }}{{x}^{\mu }}{{x}^{\nu }}=\frac{1}{2{{r}^{2}}}\left( {{\alpha }^{\mu }}{{\alpha }^{\nu }}+{{\alpha }^{\nu }}{{\alpha }^{\mu }} \right){{x}^{\mu }}{{x}^{\nu }} \\
Line 141: Line 141:
Für
Für


<math>\beta =\left( \begin{matrix}
:<math>\beta =\left( \begin{matrix}


1 & 0  \\
1 & 0  \\
Line 161: Line 161:
erfüllt werden:
erfüllt werden:


<math>\begin{align}
:<math>\begin{align}


& {{\alpha }_{r}}\beta =\left( \begin{matrix}
& {{\alpha }_{r}}\beta =\left( \begin{matrix}
Line 183: Line 183:
Es gilt:
Es gilt:


<math>\begin{align}
:<math>\begin{align}


& {{p}_{r}}=\frac{1}{r}\left( \bar{r}\bar{p}-i\hbar  \right) \\
& {{p}_{r}}=\frac{1}{r}\left( \bar{r}\bar{p}-i\hbar  \right) \\
Line 195: Line 195:
Also
Also


<math>H=\hbar c\left( \begin{matrix}
:<math>H=\hbar c\left( \begin{matrix}


0 & -1  \\
0 & -1  \\
Line 223: Line 223:
Ansatz für den Radialanteil
Ansatz für den Radialanteil


<math>\left( \begin{matrix}
:<math>\left( \begin{matrix}


{{\phi }_{a}}  \\
{{\phi }_{a}}  \\
Line 239: Line 239:
Eingesetzt in die Eigenwertgleichung für H:
Eingesetzt in die Eigenwertgleichung für H:


<math>\left( \begin{matrix}
:<math>\left( \begin{matrix}


{{\phi }_{a}}  \\
{{\phi }_{a}}  \\
Line 255: Line 255:
folgt:
folgt:


<math>\begin{align}
:<math>\begin{align}


& -\frac{\hbar c}{r}\frac{dG}{dr}-\frac{c\hbar q}{{{r}^{2}}}G+\frac{{{m}_{0}}{{c}^{2}}}{r}F+\frac{V}{r}F=E\frac{F}{r} \\
& -\frac{\hbar c}{r}\frac{dG}{dr}-\frac{c\hbar q}{{{r}^{2}}}G+\frac{{{m}_{0}}{{c}^{2}}}{r}F+\frac{V}{r}F=E\frac{F}{r} \\
Line 267: Line 267:
Also:
Also:


<math>\begin{align}
:<math>\begin{align}


& \left( E-{{m}_{0}}{{c}^{2}}-V \right)F+\hbar c\frac{dG}{dr}+\frac{c\hbar q}{r}G=0 \\
& \left( E-{{m}_{0}}{{c}^{2}}-V \right)F+\hbar c\frac{dG}{dr}+\frac{c\hbar q}{r}G=0 \\
Line 278: Line 278:


====Skalentransformation:====
====Skalentransformation:====
<math>\begin{align}
:<math>\begin{align}


& {{a}_{1}}=\frac{{{m}_{0}}{{c}^{2}}+E}{\hbar c} \\
& {{a}_{1}}=\frac{{{m}_{0}}{{c}^{2}}+E}{\hbar c} \\
Line 290: Line 290:
Führt man des weiteren ein:
Führt man des weiteren ein:


<math>\begin{align}
:<math>\begin{align}


& \rho :=ar \\
& \rho :=ar \\
Line 302: Line 302:
wodurch sich auch das Potenzial vereinfacht zu:
wodurch sich auch das Potenzial vereinfacht zu:


<math>\frac{V}{\hbar ca}=-\frac{\gamma }{\rho }</math>
:<math>\frac{V}{\hbar ca}=-\frac{\gamma }{\rho }</math>


:
:


<math>\begin{align}
:<math>\begin{align}


& \left( \frac{d}{d\rho }+\frac{q}{\rho } \right)G-\left( \frac{{{a}_{2}}}{a}-\frac{\gamma }{\rho } \right)F=0 \\
& \left( \frac{d}{d\rho }+\frac{q}{\rho } \right)G-\left( \frac{{{a}_{2}}}{a}-\frac{\gamma }{\rho } \right)F=0 \\
Line 316: Line 316:
<u>'''Randbedingung:'''</u>
<u>'''Randbedingung:'''</u>


<math>F(\rho ),G(\rho )</math>
:<math>F(\rho ),G(\rho )</math>


regulär bei <math>\rho \to 0</math>
regulär bei <math>\rho \to 0</math>


<math>F(\rho ),G(\rho )\to 0</math>
:<math>F(\rho ),G(\rho )\to 0</math>


für <math>\rho \to \infty </math>
für <math>\rho \to \infty </math>
Line 336: Line 336:
'''Asymptotisches Verhalten:'''
'''Asymptotisches Verhalten:'''


<math>\begin{align}
:<math>\begin{align}


& \rho \to \infty  \\
& \rho \to \infty  \\
Line 350: Line 350:
Weil <math>{{e}^{+\rho }}</math>
Weil <math>{{e}^{+\rho }}</math>


divergiert !
divergiert!


<math>\begin{align}
:<math>\begin{align}


& \rho \to 0 \\
& \rho \to 0 \\
Line 364: Line 364:
Ansatz:
Ansatz:


<math>\begin{align}
:<math>\begin{align}


& F(\rho )={{f}_{0}}{{\rho }^{\lambda }} \\
& F(\rho )={{f}_{0}}{{\rho }^{\lambda }} \\
Line 377: Line 377:


Es existieren nichttriviale Lösungen <math>{{f}_{0}},{{g}_{0}}</math>
Es existieren nichttriviale Lösungen <math>{{f}_{0}},{{g}_{0}}</math>
 
,
, falls <math>\left( \lambda +q \right)\left( \lambda -q \right)+{{\gamma }^{2}}={{\lambda }^{2}}-{{q}^{2}}+{{\gamma }^{2}}=0</math>
falls <math>\left( \lambda +q \right)\left( \lambda -q \right)+{{\gamma }^{2}}={{\lambda }^{2}}-{{q}^{2}}+{{\gamma }^{2}}=0</math>


Also <math>\lambda =\pm \sqrt{{{q}^{2}}-{{\gamma }^{2}}}>0</math>
Also <math>\lambda =\pm \sqrt{{{q}^{2}}-{{\gamma }^{2}}}>0</math>
Line 386: Line 386:
Ansatz:
Ansatz:


<math>\begin{align}
:<math>\begin{align}


& F(\rho )={{\rho }^{\lambda }}{{e}^{-\rho }}f\left( \rho  \right) \\
& F(\rho )={{\rho }^{\lambda }}{{e}^{-\rho }}f\left( \rho  \right) \\
Line 400: Line 400:
Die Lösung erfolgt über einen Potenzreihenansatz:
Die Lösung erfolgt über einen Potenzreihenansatz:


<math>\begin{align}
:<math>\begin{align}


& f(\rho )=\sum\limits_{k=0}^{\infty }{{{f}_{k}}{{\rho }^{k}}}\Rightarrow f\acute{\ }(\rho )=\sum\limits_{k=1}^{\infty }{k{{f}_{k}}{{\rho }^{k-1}}}=\sum\limits_{k=0}^{\infty }{(k+1){{f}_{k+1}}{{\rho }^{k}}} \\
& f(\rho )=\sum\limits_{k=0}^{\infty }{{{f}_{k}}{{\rho }^{k}}}\Rightarrow f\acute{\ }(\rho )=\sum\limits_{k=1}^{\infty }{k{{f}_{k}}{{\rho }^{k-1}}}=\sum\limits_{k=0}^{\infty }{(k+1){{f}_{k+1}}{{\rho }^{k}}} \\
Line 416: Line 416:
Koeffizientenvergleich liefert:
Koeffizientenvergleich liefert:


<math>\begin{align}
:<math>\begin{align}


& O\left( \frac{1}{\rho } \right):\left( \lambda +q \right){{g}_{0}}+\gamma {{f}_{0}}=0\quad \quad \left( \lambda -q \right){{f}_{0}}-\gamma {{g}_{0}}=0 \\
& O\left( \frac{1}{\rho } \right):\left( \lambda +q \right){{g}_{0}}+\gamma {{f}_{0}}=0\quad \quad \left( \lambda -q \right){{f}_{0}}-\gamma {{g}_{0}}=0 \\
Line 426: Line 426:
bis auf Normfaktor
bis auf Normfaktor


<math>\begin{align}
:<math>\begin{align}


& O\left( {{\rho }^{k}} \right):\left( \lambda +q+k+1 \right){{g}_{k+1}}-{{g}_{k}}+\gamma {{f}_{k+1}}-\frac{{{a}_{2}}}{a}{{f}_{k}}=0 \\
& O\left( {{\rho }^{k}} \right):\left( \lambda +q+k+1 \right){{g}_{k+1}}-{{g}_{k}}+\gamma {{f}_{k+1}}-\frac{{{a}_{2}}}{a}{{f}_{k}}=0 \\
Line 434: Line 434:
\end{align}</math>
\end{align}</math>


k=0,1,2,....  Rekursionsformel !!
k=0,1,2,....  Rekursionsformel!!


<math>\begin{align}
:<math>\begin{align}


& a\left[ \left( \lambda +q+k+1 \right){{g}_{k+1}}-{{g}_{k}}+\gamma {{f}_{k+1}}-\frac{{{a}_{2}}}{a}{{f}_{k}} \right]-{{a}_{2}}\left[ \left( \lambda -q+k+1 \right){{f}_{k+1}}-{{f}_{k}}+\gamma {{g}_{k+1}}-\frac{{{a}_{1}}}{a}{{g}_{k}} \right]=0 \\
& a\left[ \left( \lambda +q+k+1 \right){{g}_{k+1}}-{{g}_{k}}+\gamma {{f}_{k+1}}-\frac{{{a}_{2}}}{a}{{f}_{k}} \right]-{{a}_{2}}\left[ \left( \lambda -q+k+1 \right){{f}_{k+1}}-{{f}_{k}}+\gamma {{g}_{k+1}}-\frac{{{a}_{1}}}{a}{{g}_{k}} \right]=0 \\
Line 446: Line 446:
'''Verhalten für große k:'''
'''Verhalten für große k:'''


<math>ak{{g}_{k+1}}\approx {{a}_{2}}k{{f}_{k+1}}\Rightarrow {{f}_{k}}\approx \frac{a}{{{a}_{2}}}{{g}_{k}}</math>
:<math>ak{{g}_{k+1}}\approx {{a}_{2}}k{{f}_{k+1}}\Rightarrow {{f}_{k}}\approx \frac{a}{{{a}_{2}}}{{g}_{k}}</math>


Dies kann man einsetzen in
Dies kann man einsetzen in


<math>\left( \lambda +q+k+1 \right){{g}_{k+1}}-{{g}_{k}}+\gamma {{f}_{k+1}}-\frac{{{a}_{2}}}{a}{{f}_{k}}=0</math>
:<math>\left( \lambda +q+k+1 \right){{g}_{k+1}}-{{g}_{k}}+\gamma {{f}_{k+1}}-\frac{{{a}_{2}}}{a}{{f}_{k}}=0</math>


und es folgt:
und es folgt:


<math>\begin{align}
:<math>\begin{align}


& \left( k+1 \right){{g}_{k+1}}\approx 2{{g}_{k}} \\
& \left( k+1 \right){{g}_{k+1}}\approx 2{{g}_{k}} \\
Line 468: Line 468:
Falls die Potenzreihen
Falls die Potenzreihen


<math>f(\rho )=\sum\limits_{k=0}^{\infty }{{{f}_{k}}{{\rho }^{k}}},g(\rho )=\sum\limits_{k=0}^{\infty }{{{g}_{k}}{{\rho }^{k}}}</math>
:<math>f(\rho )=\sum\limits_{k=0}^{\infty }{{{f}_{k}}{{\rho }^{k}}},g(\rho )=\sum\limits_{k=0}^{\infty }{{{g}_{k}}{{\rho }^{k}}}</math>


nicht abbrechen, so divergiert <math>\begin{align}
nicht abbrechen, so divergiert <math>\begin{align}
Line 480: Line 480:
exponentiell für <math>\rho \to \infty \Rightarrow F(\rho ),G(\rho )\tilde{\ }{{e}^{\rho }}</math>
exponentiell für <math>\rho \to \infty \Rightarrow F(\rho ),G(\rho )\tilde{\ }{{e}^{\rho }}</math>


Dies ist jedoch ein Widerspruch zu den gesetzten Randbedingungen !
Dies ist jedoch ein Widerspruch zu den gesetzten Randbedingungen!


Also muss es einen Abbruch bei <math>k=n\acute{\ }\in N</math>
Also muss es einen Abbruch bei <math>k=n\acute{\ }\in N</math>
Line 486: Line 486:
geben:
geben:


<math>{{f}_{n\acute{\ }+1}}={{g}_{n\acute{\ }+1}}=0</math>
:<math>{{f}_{n\acute{\ }+1}}={{g}_{n\acute{\ }+1}}=0</math>


Setzt man dies in die Rekursionsformel ein, so folgt:
Setzt man dies in die Rekursionsformel ein, so folgt:


<math>\begin{align}
:<math>\begin{align}


& -{{g}_{n\acute{\ }}}-\frac{{{a}_{2}}}{a}{{f}_{n\acute{\ }}}=0\Rightarrow {{a}_{2}}{{f}_{n\acute{\ }}}=-a{{g}_{n\acute{\ }}} \\
& -{{g}_{n\acute{\ }}}-\frac{{{a}_{2}}}{a}{{f}_{n\acute{\ }}}=0\Rightarrow {{a}_{2}}{{f}_{n\acute{\ }}}=-a{{g}_{n\acute{\ }}} \\
Line 500: Line 500:
Diese beiden Gleichungen stimmen jedoch für alle f,g überein, da
Diese beiden Gleichungen stimmen jedoch für alle f,g überein, da


<math>\frac{{{a}_{2}}}{a}=\frac{a}{{{a}_{1}}}</math>
:<math>\frac{{{a}_{2}}}{a}=\frac{a}{{{a}_{1}}}</math>


Setzt man <math>{{a}_{2}}{{f}_{n\acute{\ }}}=-a{{g}_{n\acute{\ }}}</math>
Setzt man <math>{{a}_{2}}{{f}_{n\acute{\ }}}=-a{{g}_{n\acute{\ }}}</math>
Line 510: Line 510:
:
:


<math>\begin{align}
:<math>\begin{align}


& \frac{a\left( \lambda +q+n\acute{\ } \right)+{{a}_{2}}\gamma }{a}=-\frac{\left[ {{a}_{2}}\left( \lambda -q+n\acute{\ } \right)-a\gamma  \right]}{{{a}_{2}}} \\
& \frac{a\left( \lambda +q+n\acute{\ } \right)+{{a}_{2}}\gamma }{a}=-\frac{\left[ {{a}_{2}}\left( \lambda -q+n\acute{\ } \right)-a\gamma  \right]}{{{a}_{2}}} \\
Line 526: Line 526:
Weiter gilt:
Weiter gilt:


<math>\begin{align}
:<math>\begin{align}


& {{a}^{2}}=\frac{{{m}_{0}}^{2}{{c}^{4}}-{{E}^{2}}}{{{\hbar }^{2}}{{c}^{2}}} \\
& {{a}^{2}}=\frac{{{m}_{0}}^{2}{{c}^{4}}-{{E}^{2}}}{{{\hbar }^{2}}{{c}^{2}}} \\
Line 536: Line 536:
Löst man dies nach den exakten Energieeigenwerten, die sich damit ergeben, also nach E auf, so erhält man die Feinstrukturformel:
Löst man dies nach den exakten Energieeigenwerten, die sich damit ergeben, also nach E auf, so erhält man die Feinstrukturformel:


<math>E=\frac{{{m}_{0}}{{c}^{2}}}{\sqrt{1+{{\left( \frac{\gamma }{\lambda +n\acute{\ }} \right)}^{2}}}}</math>
:<math>E=\frac{{{m}_{0}}{{c}^{2}}}{\sqrt{1+{{\left( \frac{\gamma }{\lambda +n\acute{\ }} \right)}^{2}}}}</math>


Mit der Feinstrukturkonstanten
Mit der Feinstrukturkonstanten


<math>\gamma \approx \frac{1}{137}</math>
:<math>\gamma \approx \frac{1}{137}</math>


<math>\begin{align}
:<math>\begin{align}


& \lambda =\sqrt{q} \\
& \lambda =\sqrt{q} \\
Line 552: Line 552:
\end{align}</math>
\end{align}</math>


<math>\begin{align}
:<math>\begin{align}


& \lambda =\sqrt{{{q}^{2}}-{{\gamma }^{2}}}=\sqrt{{{\left( j+\frac{1}{2} \right)}^{2}}-{{\gamma }^{2}}} \\
& \lambda =\sqrt{{{q}^{2}}-{{\gamma }^{2}}}=\sqrt{{{\left( j+\frac{1}{2} \right)}^{2}}-{{\gamma }^{2}}} \\
Line 563: Line 563:


entwickelt man die Energieeigenwerte nach der Feinstrukturkonstanten bis <math>O\left( {{\gamma }^{4}} \right)</math>
entwickelt man die Energieeigenwerte nach der Feinstrukturkonstanten bis <math>O\left( {{\gamma }^{4}} \right)</math>
,
so folgt:


, so folgt:
:<math>E={{m}_{0}}{{c}^{2}}\left[ 1-\frac{1}{2}{{\left( \frac{\gamma }{\lambda +n\acute{\ }} \right)}^{2}}+\frac{3}{8}{{\left( \frac{\gamma }{\lambda +n\acute{\ }} \right)}^{4}}+O\left( {{\gamma }^{6}} \right) \right]</math>
 
<math>E={{m}_{0}}{{c}^{2}}\left[ 1-\frac{1}{2}{{\left( \frac{\gamma }{\lambda +n\acute{\ }} \right)}^{2}}+\frac{3}{8}{{\left( \frac{\gamma }{\lambda +n\acute{\ }} \right)}^{4}}+O\left( {{\gamma }^{6}} \right) \right]</math>


mit
mit


<math>\lambda \left( \gamma  \right)=|q|\sqrt{1-{{\left( \frac{\gamma }{q} \right)}^{2}}}=|q|\left[ 1-\frac{1}{2}{{\left( \frac{\gamma }{q} \right)}^{2}} \right]+O\left( {{\gamma }^{4}} \right)</math>
:<math>\lambda \left( \gamma  \right)=|q|\sqrt{1-{{\left( \frac{\gamma }{q} \right)}^{2}}}=|q|\left[ 1-\frac{1}{2}{{\left( \frac{\gamma }{q} \right)}^{2}} \right]+O\left( {{\gamma }^{4}} \right)</math>


<math>\begin{align}
:<math>\begin{align}


& {{\left( \frac{1}{\lambda +n\acute{\ }} \right)}^{2}}=\frac{1}{{{\left[ n\acute{\ }+|q|-\frac{1}{2}\left( \frac{{{\gamma }^{2}}}{\left| q \right|} \right) \right]}^{2}}}+O\left( {{\gamma }^{4}} \right) \\
& {{\left( \frac{1}{\lambda +n\acute{\ }} \right)}^{2}}=\frac{1}{{{\left[ n\acute{\ }+|q|-\frac{1}{2}\left( \frac{{{\gamma }^{2}}}{\left| q \right|} \right) \right]}^{2}}}+O\left( {{\gamma }^{4}} \right) \\
Line 582: Line 582:
\end{align}</math>
\end{align}</math>


Setzt man dies in die exakten Energieeigenwerte E ein , so folgt:
Setzt man dies in die exakten Energieeigenwerte E ein, so folgt:
<math>\begin{align}
:<math>\begin{align}
& E={{m}_{0}}{{c}^{2}}\left[ 1-\left( \frac{{{\gamma }^{2}}}{2{{n}^{2}}} \right)-\left( \frac{{{\gamma }^{4}}}{2{{n}^{3}}} \right)\left( \frac{1}{j+\frac{1}{2}}-\frac{3}{4n} \right)+O\left( {{\gamma }^{6}} \right) \right] \\
& E={{m}_{0}}{{c}^{2}}\left[ 1-\left( \frac{{{\gamma }^{2}}}{2{{n}^{2}}} \right)-\left( \frac{{{\gamma }^{4}}}{2{{n}^{3}}} \right)\left( \frac{1}{j+\frac{1}{2}}-\frac{3}{4n} \right)+O\left( {{\gamma }^{6}} \right) \right] \\
& n=1,2,3 \\
& n=1,2,3 \\
Line 591: Line 591:


<u>'''Diskussion'''</u>
<u>'''Diskussion'''</u>
<math>O\left( {{\gamma }^{0}} \right):E={{m}_{0}}{{c}^{2}}</math>
:<math>O\left( {{\gamma }^{0}} \right):E={{m}_{0}}{{c}^{2}}</math>
Ruheenergie
Ruheenergie
<math>O\left( {{\gamma }^{2}} \right):\Delta {{E}^{(2)}}=-{{m}_{0}}{{c}^{2}}\left( \frac{{{\gamma }^{2}}}{2{{n}^{2}}} \right)=-\frac{{{R}_{H}}}{{{n}^{2}}}</math>
:<math>O\left( {{\gamma }^{2}} \right):\Delta {{E}^{(2)}}=-{{m}_{0}}{{c}^{2}}\left( \frac{{{\gamma }^{2}}}{2{{n}^{2}}} \right)=-\frac{{{R}_{H}}}{{{n}^{2}}}</math>
nicht relativistisches, entartetes Energiespektrum
nicht relativistisches, entartetes Energiespektrum
<math>O\left( {{\gamma }^{4}} \right):\Delta {{E}^{(4)}}=-{{m}_{0}}{{c}^{2}}\left( \frac{{{\gamma }^{4}}}{2{{n}^{3}}} \right)\left( \frac{1}{j+\frac{1}{2}}-\frac{3}{4n} \right)</math>
:<math>O\left( {{\gamma }^{4}} \right):\Delta {{E}^{(4)}}=-{{m}_{0}}{{c}^{2}}\left( \frac{{{\gamma }^{4}}}{2{{n}^{3}}} \right)\left( \frac{1}{j+\frac{1}{2}}-\frac{3}{4n} \right)</math>
Feinstruktur- Aufspaltung. Eine Aufhebung der j-Entartung durch Spin- Bahn- Kopplung.
Feinstruktur- Aufspaltung. Eine Aufhebung der j-Entartung durch Spin- Bahn- Kopplung.
Dabei bleibt die Freiheit der Ausrichtung der Achse des magnetischen Moments, also die <math>2(2j+1)</math>
Dabei bleibt die Freiheit der Ausrichtung der Achse des magnetischen Moments, also die <math>2(2j+1)</math>
- fache <math>{{m}_{j}}</math>
- fache <math>{{m}_{j}}</math>
- Entartung+ Parität !
- Entartung+ Parität!
====Spektroskopische Beziehung der Feinstrukturterme: <math>n{{l}_{j}}</math>====
====Spektroskopische Beziehung der Feinstrukturterme: <math>n{{l}_{j}}</math>====
<math>n=1:\quad j=\frac{1}{2}:\ 1{{s}_{\frac{1}{2}}}</math>
:<math>n=1:\quad j=\frac{1}{2}:\ 1{{s}_{\frac{1}{2}}}</math>
<math>\begin{array}{*{35}{l}}
:<math>\begin{array}{*{35}{l}}
   {} & n=2:\quad j=\frac{1}{2}:\ 2{{s}_{\frac{1}{2}}}\quad 2{{p}_{\frac{1}{2}}}\quad \quad \quad \quad \quad \quad \quad n\overset{\acute{\ }}{\mathop{\ }}\,=1  \\
   {} & n=2:\quad j=\frac{1}{2}:\ 2{{s}_{\frac{1}{2}}}\quad 2{{p}_{\frac{1}{2}}}\quad \quad \quad \quad \quad \quad \quad n\overset{\acute{\ }}{\mathop{\ }}\,=1  \\
   {} & \quad \quad \quad \,j=\frac{3}{2}:\quad \quad \quad 2{{p}_{\frac{3}{2}}}\quad \quad \quad \quad \quad \quad \quad n\overset{\acute{\ }}{\mathop{\ }}\,=0  \\
   {} & \quad \quad \quad \,j=\frac{3}{2}:\quad \quad \quad 2{{p}_{\frac{3}{2}}}\quad \quad \quad \quad \quad \quad \quad n\overset{\acute{\ }}{\mathop{\ }}\,=0  \\
\end{array}</math>
\end{array}</math>


n´=0
n´=0.
..
.

Latest revision as of 00:35, 13 September 2010


{{#set:Urheber=Prof. Dr. E. Schöll, PhD|Inhaltstyp=Script|Kapitel=7|Abschnitt=5}} Kategorie:Quantenmechanik __SHOWFACTBOX__


In einem rotationssymmetrischen Potenzial haben wir als Dirac- Hamiltonian:

H=(cα¯p¯+m0c2β+V(r))pr:=1r(r¯p¯i)αr:=1rα¯r¯Q:=β(σ¯~L¯+)

Dabei sind pr,αr,Q

hermitesche Operatoren

Man kann den Hamilton- Operator schreiben als:

H=(cαrpr+icrαrβQ+m0c2β+V(r))

Beweis:

αrpr+irαrβQ=αr[1r(r¯p¯i)+irβ2(σ¯~L¯+)]β2=1=αrr(r¯p¯+iσ¯~L¯)=1r2[(α¯r¯)(r¯p¯)+i(α¯r¯)(σ¯~L¯)]i(α¯r¯)(σ¯~L¯)=i(α¯r¯)(r¯p¯)ir2(α¯p¯)αrpr+irαrβQ=1r2[(α¯r¯)(r¯p¯)+i(α¯r¯)(σ¯~L¯)]=α¯p¯

Es gilt weiter:

[Q,H]=0

.

Somit existieren gemeinsame Eigenzustände zu H

und Q

Eigenwerte von Q

(Q)2=β(σ¯~L¯+)β(σ¯~L¯+)=β2(σ¯~L¯+)2[β,σ¯~]=0=([1,σ¯~][1,σ¯~])β2=1(Q)2=(σ¯~L¯)(σ¯~L¯)+2(σ¯~L¯)+2(σ¯~L¯)(σ¯~L¯)=L2+iσ¯~(L¯×L¯)(L¯×L¯)=iL¯(σ¯~L¯)(σ¯~L¯)=L2σ¯~(L¯)

Somit:

(Q)2=L2+σ¯~L¯+2=(L¯+2σ¯~)2+24mit(L¯+2σ¯~)2=L2+σ¯~L¯+24σ¯~2σ¯~2=3(L¯+2σ¯~)=J¯

Schließlich also

(Q)2=J¯2+24

Die Eigenwerte von J¯2

sind jedoch bekannt, nämlich j(j+1)

mit j=l±s=12,32,...

(Q)2|j=(2j(j+1)+24)|j=2(j+12)2|j(j+12)2:=q2

Somit:

(Q)|j=(q)|jq=±1,±2,...

Es bleibt das radiale Eigenwertproblem für

H=(cαrpr+icrαrβQ+m0c2β+V(r))

Geeignete Darstellung für αr

(αr)2=1r2(α¯r¯)(α¯r¯)=1r2αμανxμxν=12r2(αμαν+αναμ)xμxν(αμαν+αναμ)=2δμν12r22xμxμ=r2r2=1αrβ+βαr=1r(α¯β+βα¯)r¯(α¯β+βα¯)=01r(α¯β+βα¯)r¯=0

Für

β=(1001)

kann dies durch die Darstellung αr=(0ii0)

mit αr=αr+

erfüllt werden:

αrβ=(0ii0)βαr=(0ii0)

Es gilt:

pr=1r(r¯p¯i)r¯p¯=irrpr=1r(irri)=i(r+1r)

Also

H=c(0110)(r+1r)cqr(0110)+m0c2(1001)+V(1001)

Ansatz für den Radialanteil

(ϕaϕb)~1r(F(r)G(r))

Eingesetzt in die Eigenwertgleichung für H:

(ϕaϕb)~1r(F(r)G(r))

folgt:

crdGdrcqr2G+m0c2rF+VrF=EFrcrdFdrcqr2Fm0c2rG+VrG=EGrV=e24πε01r

Also:

(Em0c2V)F+cdGdr+cqrG=0(E+m0c2V)GcdFdr+cqrF=0V=e24πε01r

Skalentransformation:[edit | edit source]

a1=m0c2+Eca2=m0c2Eca=a1a2=m02c4E2c

Führt man des weiteren ein:

ρ:=arγ:=e24πε0c1137

Also einen skalierten Radius und die Feinstrukturkonstante,

wodurch sich auch das Potenzial vereinfacht zu:

Vca=γρ
(ddρ+qρ)G(a2aγρ)F=0(ddρqρ)F(a1a+γρ)G=0

Randbedingung:

F(ρ),G(ρ)

regulär bei ρ0

F(ρ),G(ρ)0

für ρ

Betrachte |E|<m0c2a1,a2>0aR

also gebundene Zustände

Asymptotisches Verhalten:

ρG´=a2aFF´=a1aGG´´=G,F´´=FG=eρ=F=G=eρ

Weil e+ρ

divergiert!

ρ0G´+qρG+γρF=0F´qρFγρG=0

Ansatz:

F(ρ)=f0ρλG(ρ)=g0ρλ(λ+q)g0+γf0=0(λq)f0γg0=0

Es existieren nichttriviale Lösungen f0,g0 ,

falls (λ+q)(λq)+γ2=λ2q2+γ2=0

Also λ=±q2γ2>0

und regulär bei ρ0

Ansatz:

F(ρ)=ρλeρf(ρ)G(ρ)=ρλeρg(ρ)g´g+λ+qρg(a2aγρ)f=0f´f+λqρf(a1a+γρ)g=0

Die Lösung erfolgt über einen Potenzreihenansatz:

f(ρ)=k=0fkρkf´(ρ)=k=1kfkρk1=k=0(k+1)fk+1ρkg(ρ)=k=0gkρkg´(ρ)=k=1kgkρk1f(ρ)ρ=k=0fkρk1=f0ρ+k=0fk+1ρk

usw... wird dies ebenfalls für g´(ρ),g(ρ)ρ

aufgestellt

Koeffizientenvergleich liefert:

O(1ρ):(λ+q)g0+γf0=0(λq)f0γg0=0f0,g0

bis auf Normfaktor

O(ρk):(λ+q+k+1)gk+1gk+γfk+1a2afk=0(λq+k+1)fk+1fk+γgk+1a1agk=0

k=0,1,2,.... Rekursionsformel!!

a[(λ+q+k+1)gk+1gk+γfk+1a2afk]a2[(λq+k+1)fk+1fk+γgk+1a1agk]=0[a(λ+q+k+1)+a2γ]gk+1=[a2(λq+k+1)aγ]fk+1

Verhalten für große k:

akgk+1a2kfk+1fkaa2gk

Dies kann man einsetzen in

(λ+q+k+1)gk+1gk+γfk+1a2afk=0

und es folgt:

(k+1)gk+12gkgk+1gk2k+1gk+12k+1(k+1)!g0g(ρ)~e2ρf(ρ)~e2ρ

Falls die Potenzreihen

f(ρ)=k=0fkρk,g(ρ)=k=0gkρk

nicht abbrechen, so divergiert F(ρ)=ρλeρf(ρ)G(ρ)=ρλeρg(ρ)

exponentiell für ρF(ρ),G(ρ)~eρ

Dies ist jedoch ein Widerspruch zu den gesetzten Randbedingungen!

Also muss es einen Abbruch bei k=n´N

geben:

fn´+1=gn´+1=0

Setzt man dies in die Rekursionsformel ein, so folgt:

gn´a2afn´=0a2fn´=agn´fn´a1agn´=0afn´=a1gn´

Diese beiden Gleichungen stimmen jedoch für alle f,g überein, da

a2a=aa1

Setzt man a2fn´=agn´

in [a(λ+q+k+1)+a2γ]gk+1=[a2(λq+k+1)aγ]fk+1

ein, so folgt mit k+1=n´

a(λ+q+n´)+a2γa=[a2(λq+n´)aγ]a2λ+q+n´+a2aγ+λq+n´+aa2γ=02a(λ+n´)=(a2a2a2)γ=2Ecγa2a2=a1a2(λ+n´)2=E22c2γ2

Weiter gilt:

a2=m02c4E22c2(m02c4E2)(λ+n´)2=E2γ2

Löst man dies nach den exakten Energieeigenwerten, die sich damit ergeben, also nach E auf, so erhält man die Feinstrukturformel:

E=m0c21+(γλ+n´)2

Mit der Feinstrukturkonstanten

γ1137
λ=qa2=m02c4E22c2(m02c4E2)(λ+n´)2=E2γ2
λ=q2γ2=(j+12)2γ2j=12,32,...,n´N0j=l±s

entwickelt man die Energieeigenwerte nach der Feinstrukturkonstanten bis O(γ4) ,

so folgt:
E=m0c2[112(γλ+n´)2+38(γλ+n´)4+O(γ6)]

mit

λ(γ)=|q|1(γq)2=|q|[112(γq)2]+O(γ4)
(1λ+n´)2=1[n´+|q|12(γ2|q|)]2+O(γ4)n=n´+|q|n´=0,1,2,...|q|=j+12=1,2,....(1λ+n´)2=1n2[112(γ2|q|n)]2+O(γ4)=1n2[1+(γ2|q|n)]+O(γ4)=1n2+(γ2|q|n3)+O(γ4)|q|=j+12=l±s+12

Setzt man dies in die exakten Energieeigenwerte E ein, so folgt:

E=m0c2[1(γ22n2)(γ42n3)(1j+1234n)+O(γ6)]n=1,2,3j=12,32,...,n12,wegenn=n´+j+12j=l±s

Diskussion

O(γ0):E=m0c2

Ruheenergie

O(γ2):ΔE(2)=m0c2(γ22n2)=RHn2

nicht relativistisches, entartetes Energiespektrum

O(γ4):ΔE(4)=m0c2(γ42n3)(1j+1234n)

Feinstruktur- Aufspaltung. Eine Aufhebung der j-Entartung durch Spin- Bahn- Kopplung. Dabei bleibt die Freiheit der Ausrichtung der Achse des magnetischen Moments, also die 2(2j+1) - fache mj - Entartung+ Parität!

Spektroskopische Beziehung der Feinstrukturterme: nlj[edit | edit source]

n=1:j=12:1s12
n=2:j=12:2s122p12n´=1j=32:2p32n´=0

n´=0. .